You are on page 1of 6

Bibliothèque d’exercices Énoncés

L1 Feuille n◦ 9

Propriétés de R

1 Les rationnels Q
Exercice 1 1. Démontrer que si r ∈ Q et x 6∈ Q alors r + x 6∈ Q et si r 6= 0 r.x 6∈ Q.

2. Montrer que 2 6∈ Q,
3. En déduire : entre 2 nombres rationnels il y a toujours un nombre irrationnel. (On pourra
utiliser la propriété : pour tout réel a > 0, il existe un entier n tel que n > a.)
Pn
Exercice 2 Soit p(x) = i=0 ai xi . On suppose que tous les ai sont des entiers.
1. Montrer que si p a une racine rationnelle αβ alors α divise a0 et β divise an .
√ √
2. On considère le nombre 2 + 3. En calculant son carré, montrer que ce carré est racine
d’un polynôme de degré 2. En déduire, à l’aide du résultat précédent qu’il n’est pas
rationnel.
p
Exercice 3 1. Soit Nn = 0, 1997 1997 . . . 1997 (n fois). Mettre Nn sous la forme q
avec
p, q ∈ N∗ .
2. Soit M = 0, 1997 1997 1997 . . . . . . Donner le rationnel dont l’écriture décimale est M .
3. Même question avec : P = 0, 11111 . . .+0, 22222 . . .+0, 33333 . . .+0, 44444 . . .+0, 55555 . . .+
0, 66666 . . . + 0, 77777 . . . + 0, 88888 . . . + 0, 99999 . . .
ln 3
Exercice 4 Montrer que ln 2
est irrationnel.

2 Maximum, minimum, borne supérieure...


Exercice 5 Le maximum de 2 nombres x, y (c’est-à-dire le plus grand des 2) est noté max(x, y).
De même on notera min(x, y) le plus petit des 2 nombres x, y. Démontrer que :

x + y + |x − y| x + y − |x − y|
max(x, y) = et min(x, y) = .
2 2
Trouver une formule pour max(x, y, z).

Exercice 6 Déterminer la borne supérieure et inférieure (éventuellement infinies) de : A =


{un , n ∈ N} en posant un = 2n si n est pair et un = 2−n sinon.

Exercice 7 Déterminer (s’ils existent) : les majorants, les minorants, la borne supérieure, la
borne inférieure, le plus grand élément, le plus petit élément des ensembles suivants :
 
n 1 ∗
[0, 1] ∩ Q , ]0, 1[∩Q , N , (−1) + , n ∈ N .
n

1
Exercice 8 Soient A et B deux parties bornées de R. On note A+B = {a+b | (a, b) ∈ A×B}.
1. Montrer que sup A + sup B est un majorant de A + B.
2. Montrer que sup(A + B) = sup A + sup B.

Exercice 9 Soit A et B deux parties bornées de R. Vrai ou faux ?


1. A ⊂ B ⇒ sup A 6 sup B,
2. B ⊂ A ⇒ inf A 6 inf B,
3. sup A ∪ B = max(sup A, sup B),
4. sup(A + B) < sup A + sup B,
5. sup(−A) = − inf A,
6. sup A + inf B 6 sup(A + B).

3 Divers
Exercice 10 Si a et b sont des réels > 0, montrer que :
√ √ √
a + b 6 2 a + b.

Exercice 11 Soit f : R → R croissante telle que ∀(x, y) ∈ R2 f (x + y) = f (x) + f (y). Montrer


que
1. ∀n ∈ N f (n) = nf (1).
2. ∀n ∈ Z f (n) = nf (1).
3. ∀q ∈ Q f (q) = qf (1).
4. ∀x ∈ R f (x) = xf (1) (on pourra utiliser la densité de Q dans R pour encadrer x par
des rationnels de plus en plus proches de x).

2
Bibliothèque d’exercices Indications
L1 Feuille n◦ 9

Propriétés de R

Indication 1 1. Raisonner par l’absurde.



2. Raisonner par l’absurde en écrivant 2 = pq avec p et q premiers entre eux. Puis essayer
de montrer que p et q sont tous les deux pairs.
3. Utiliser les deux questions précédentes.

Indication 2 1. Calculer β n p( αβ ) et utiliser le théorème de Gauss.


2. Utiliser la première question avec p(x) = (x2 − 5)5 − 24.

Indication 3 1. Mutiplier Nn par une puissance de 10 suffisament grande pour obtenir un


nombre entier.
2. Mutiplier N par une puissance de 10 suffisament grande (pas trop grande) puis soustraire
N pour obtenir un nombre entier.

Indication 4 C’est le même type de démonstration que pour prouver que 2 n’est pas ra-
tionnel.

Indication 6 sup A = +∞, inf A = 0.

Indication 10 Élever l’inégalité au carré.

Indication 11 1. f (2) = f (1 + 1) = · · · , faire une récurrence.


2. f ((−n) + n) = · · · .
3. Si q = ab , calculer f ( ab + a
b
+ · · · + ab ) avec b termes dans cette somme.
4. Pour x ∈ R fixé, prendre une suite de rationnels qui croit vers x, et une autre qui décroit
vers x.

1
Bibliothèque d’exercices Corrections
L1 Feuille n◦ 9

Propriétés de R

p
Correction 1 1. Soit r = q
∈ Q et x ∈
/ Q. Par l’absurde supposons que r + x ∈ Q alors
0 0 0 0
il existe deux entiers p , q tels que r + x = pq0 . Donc x = pq0 − pq = qp qq−pq
0 0
0 ∈ Q ce qui est
absurde car x ∈ / Q.
0 0
De la même façon si rx ∈ Q alors rx = pq0 Et donc x = pq0 pq . Ce qui est absurde.
√ √
2. Supposons que 2 ∈ Q alors il existe deux entiers p, q tels que 2 = pq . De plus nous
pouvons supposer que la fraction est irréductible (p et q sont premiers entre eux). En
élevant l’égalité au carré nous obtenons q 2 × 2 = p2 . Donc p2 est un nombre pair, cela
implique que p est un nombre pair (si vous n’êtes pas convaincu écrivez la contraposée
“p impair ⇒ p2 impair”). Donc p = 2 × p0 avec p0 ∈ N, d’où p2 = 4 × p0 2 . Nous obtenons
q 2 = 2 × p0 2 . Nous en déduisons maintenant que q 2 est pair et comme ci-dessus que q est
pair. Nous obtenons ainsi une contradiction car p et q étant√tous les deux pairs la fraction
p
q
n’est pas irréductible et aurait pu être simplifier. Donc 2 ∈ / Q.
0 0
√ 0
3. Soient√ r, r deux rationnels avec r < r . Notons a = 2(r − r). Choisissons n ∈ N tel que
n > 2. Et posons
a
x=r+ .
n
√ 0
D’une part x ∈]r, r [ et d’après les deux premières questions 2 r n−r ∈
0

/ Q. Et donc x est
0
un nombre irrationnel compris entre r et r .
 i
1. Soit αβ ∈ Q avec α ∧ β = 1. Pour p( αβ ) = 0, alors ni=1 ai αβ = 0. Après
P
Correction 2
multiplication par β n nous obtenons l’égalité suivante :

an αn + an−1 αn−1 β + · · · + a1 αβ n−1 + a0 β n .

En factorisant les derniers termes de cette somme par β, nous écrivons an αn +βq = 0. Ceci
entraı̂ne que β divise an αn , mais comme β et αn sont premier entre eux (car α ∧ β = 1)
alors par le théorème de Gauss β divise an . De même en factorisant les premiers termes de
la somme ci-dessus par α nous obtenons αq 0 + a0 β n = 0 et par un raisonnement similaire
α divise a0 .
√ √ √ √ 2
2. Notons γ = 2 + 3. Alors γ 2 = 5 + 2 2 3 Et donc (γ 2 − 5) = 4 × 2 × 3, Nous
choisissons p(x) = (x2 − 5)5 − 24, qui s’écrit aussi p(x) = x4 − 10x2 + 1. Vu notre choix de
p, nous avons p(γ) = 0. Si nous supposons que γ est rationnel, alors γ = αβ et d’après la
première question α divise le terme constant de p, c’est-à-dire 1. Donc α = ±1. De même
β divise le coefficient du terme de plus au degré de p, donc β divise 1, soit β = 1. Ainsi
γ = ±1, ce qui est évidemment absurde !

Correction 3 1. Soit p = 2001 2001 . . . 2001 et q = 10000 0000 . . . 0000 = 104n . Alors
Nn = pq .
2. Remarquons que 10 000 × M = 2001, 2001 2001 . . . Alors 10 000 × M − M = 2001 ; donc
2001
9999 × M = 2001 d’où M = 9999 .

1
3. 0, 111 . . . = 91 , 0, 222 . . . = 92 , etc. D’où P = 1
9
+ 29 + · · · + 9
9
= 1+2+···+9
9
= 45
9
= 5.

Correction 4 Par l’absurde supposons que ln 3


ln 2
est un rationnel. Il s’écrit pq avec p > 0, q > 0
des entiers. On obtient q ln 3 = p ln 2. En prenant l’exponentielle : exp(q ln 3) = exp(p ln 2) soit
3q = 2p . Si p > 1 alors 2 divise 3q , ce qui est absurde. Donc p = 0 ou p = 1. Donc 3q = 2
ou 3q = 1. La seule solution possible est p = 0, q = 0. Ce qui contredit q 6= 0. Donc ln 3
ln 2
est
irrationnel.

Correction 5 Explicitons la formule pour max(x, y). Si x > y, alors |x − y| = x − y donc


1
2
(x + y + |x − y|) = 12 (x + y + x − y) = x. De même si x 6 y, alors |x − y| = −x + y donc
1
2
(x + y + |x − y|) = 12 (x + y − x + y) = y. 
Pour 3 élément, nous avons max(x, y, z) = max max(x, y), z , donc d’après les formules pour
2 éléments :
max(x, y) + z + | max(x, y) − z|
max(x, y, z) =
2 1
1
(x + y + |x − y|) + z + (x + y + |x − y|) − z
= 2 2
.
2
Correction 6 (u2k )k tend vers +∞ et donc le seul majorant de A est +∞ et donc sup A =
+∞. D’autre part toutes les valeurs de (un ) sont positives et (u2k+1 )k tend vers 0, donc inf A = 0.

Correction 7 1. [0, 1] ∩ Q. Les majorants : [1, +∞[. Les minorants : ] − ∞, 0]. La borne
supérieure : 1. La borne inférieure : 0. Le plus grand élément : 1. Le plus petit élément 0.
2. ]0, 1[∩Q. Les majorants : [1, +∞[. Les minorants : ] − ∞, 0]. La borne supérieure : 1. La
borne inférieure : 0. Il nexiste pas de plus grand élément ni de plus petit élément.
3. N. Pas de majorants, pas de borne supérieure, ni de plus grand élément. Les minorants :
] − ∞, 0]. La borne inférieure : 0. Le plus petit élément : 0.
n o
4. (−1)n + n12 , n ∈ N∗ . Les majorants : [ 54 , +∞[. Les minorants : ] − ∞, −1]. La borne
supérieure : 54 . La borne inférieure : −1. Le plus grand élément : 45 . Pas de plus petit
élément.

Correction 8 1. Soient A et B deux parties bornées de R. On sait que Sup A est un


majorant de A, c’est à dire, ∀a ∈ A, a 6 Sup A. De même, ∀b ∈ B, b 6 Sup B. On veut
montrer que Sup A + Sup B est un majorant de A + B. Soit donc x ∈ A + B. Cela signifie
que x est de la forme a + b pour un a ∈ A et un b ∈ B. Or a 6 Sup A, et b 6 Sup B, donc
x = a + b 6 Sup A + Sup B. Comme ce raisonnement est valide pour tout x ∈ A + B cela
signifie que Sup A + Sup B est un majorant de A + B.
2. On veut montrer que, quel que soit ε > 0, Sup A + Sup B − ε n’est pas un majorant de
A + B. On prend donc un ε > 0 quelconque, et on veut montrer que Sup A + Sup B − ε
ne majore pas A + B. On s’interdit donc dans la suite de modifier ε. Comme Sup A est
le plus petit des majorants de A, Sup A − ε/2 n’est pas un majorant de A. Cela signifie
qu’il existe un élément a de A tel que a > Sup A − ε/2. Attention : Sup A − ε n’est
pas forcément dans A. Sup A non plus. Et il n’est pas non plus vrai que ∀a ∈ A a >
Sup A − ε/2. On ne choisit donc pas ce a. De la même manière, il existe b ∈ B tel que
b > Sup B − ε/2. Or l’élément x défini par x = a + b est un élément de A + B, et il
vérifie x > (Sup A − ε/2) + (Sup B − ε/2) = Sup A + Sup B − ε. Ceci implique que
Sup A + Sup B − ε n’est pas un majorant de A + B.

2
3. Sup A + Sup B est un majorant de A + B d’après la partie 1. Mais, d’après la partie 2., dès
qu’on prend un ε > 0, SupA+SupB−ε n’est pas un majorant de A+B. Donc SupA+SupB
est bien le plus petit des majorants de A + B, i.e. Sup (A + B) = Sup A + Sup B.

Correction 9 1. Vrai.
2. Vrai.
3. Vrai.
4. Faux. L’égalité peut ne pas être stricte.
5. Vrai.
6. Vrai.

Correction 10
√ √ √
a+ b62 a+b
√ √
⇔ ( a + b)2 6 2(a + b)

car les termes sont positifs, et la fonction x 7→ x2 est croissante sur R+ .


√ √
⇔ a + b + 2 a b 6 2(a + b)
√ √
⇔a+b−2 a b>0
√ √
⇔ ( a − b)2 > 2.

La denière proposition est toujours vraie, et donc par équivalence, nous obtenons l’inégalité
recherchée.

Correction 11 1. Calculons d’abord f (0). f (1) = f (1 + 0) = f (1) + f (0) Donc f (0) = 0.


Montrons le résultat demandé par récurrence : pour n = 1, nous avons bien f (1) =
1 × f (1). Si f (n) = nf (1) alors f (n + 1) = f (n) + f (1) = nf (1) + f (1) = (n + 1)f (1).
2. 0 = f (0) = f (−1 + 1) = f (−1) + f (1). Donc f (−1) = −f (1). Puis comme ci-dessus
f (−n) = nf (−1) = −nf (1).
3. Soit q = ab . Alors f (a) = f ( ab + ab + · · · + ab ) = f ( ab ) + · · · + f ( ab ) (b termes dans cette
somme). Donc f (a) = bf ( ab ). Soit af (1) = bf ( ab ). Ce qui s’écrit aussi f ( ab ) = ab f (1).
4. Soit x ∈ R Soit (αi ) une suite croissante de rationnels qui tend vers x. Soit (βi ) une suite
décroissante de rationnels qui tend vers x :

α1 6 α2 6 α3 6 . . . 6 x 6 · · · 6 β2 6 β1 .

Alors comme αi 6 x 6 βi et que f est croissante nous avons f (αi ) 6 f (x) 6 f (βi ).
D’après la question précédent cette inéquation devient : αi f (1) 6 f (x) 6 βi f (1). Comme
(αi ) et (βi ) tendent vers x. Par le théorème des “gendarmes” nous obtenons en passant à
la limite : xf (1) 6 f (x) 6 xf (1). Soit f (x) = xf (1).

You might also like